Skip to Main Content

PrepTest 79, Game 2, Question 12



If J is assigned to area two, then which one of the following must be true? So, if we drop J in 2, what logically follows, or what has to be true? So since we can learn a lot about the placement of the other variables based on the placement of O, let's see if we can drop O in 2 since it's directly related to the placement of J. So if we put O in 2, that means J and K must also go in area 2, since whenever O's in 2, J and K must be together, and the question stem tells us that J goes in 2.

This is not going to work. So when we do this, then we need to put L in 3 in order to meet this requirement that L be paired with K or M, and then we're gonna have to drop P here as well, since P can't go into 1, and there's no room for it in 2. So this does not work. It doesn't create a valid arrangement.

What do we know about O though? O cannot go in 1, so the only options for O is for it to go into 2 or 3. So that's gonna impact our answer choice here. Now that we know that O can't go into 2, we know that it must go into 3, which also means that J and K cannot be paired together, since O is going to be in 3.

So that leads us to our correct answer, which is A, K is assigned to area 1. So if O has to go in area 3, and J, we're already told that J is in area 2, we need to separate J and K. So that means that K must go into area 1. So this is our correct answer, when we put J in 2, it must be true that K goes into area 1.

Now we have P and L. So L can go either in area 1, or in area 3, no problem, and P can do the same, so it can go in area 2, or area 3, no problems there. But, K must be in 1, since we need to separate J and K, and we need to be thoughtful about our LM or LK situation here. So if we put K in 3, we're not gonna be able to appropriately place L.

So A is our correct answer.

Read full transcript